LSAT and Law School Admissions Forum

Get expert LSAT preparation and law school admissions advice from PowerScore Test Preparation.

 Administrator
PowerScore Staff
  • PowerScore Staff
  • Posts: 8916
  • Joined: Feb 02, 2011
|
#24421
Complete Question Explanation

Flaw in the Reasoning. The correct answer choice is (C)

To understand the reasoning flaw in the argument, consider the logical structure of its premises and the conclusion:
  • Premise (1): If the media conveyed a range of opinions narrower than that found among consumers, its power would be too great (N ..... :arrow: ..... PG).

    Premise (2): The media does not convey a narrower range of opinions (N)

    Conclusion: The media does not have too much power to impose its opinions on the public (PG)
You should notice that the conclusion contains a Mistaken Negation: just because the media does not convey a narrower range of opinions than that found among consumers, this does not automatically mean that its powers are properly limited. A quick look at the five answer choices reveals that only answer choice (C) contains the description of Mistaken Negation. Answer choice (C) is therefore correct.

Answer choice (A): Since the author does not attack the critics’ character or motives in attempting to discredit their position, there is no Source Argument flaw in the reasoning. This answer choice is incorrect.

Answer choice (B): For the author to take something for granted, she must be making an unwarranted (but unstated) assumption that the statement in question is a true fact. Here, the statement that the media purveys an appropriate range of opinions is a given: indeed, it is the central premise of the argument and not an unstated assumption. This answer choice is therefore incorrect.

Answer choice (C): This is the correct answer choice. See discussion above.

Answer choice (D): While the exposure of popular opinion in the media is a central issue in this argument, the author does not rely on popular opinion to substantiate her conclusion. This answer choice is incorrect.

Answer choice (E): The author never explicitly or implicitly suggest that it is desirable for a wide range of popular opinions to receive media exposure; she merely stated that the exposure of popular opinion in media is as wide as the range of opinion found among consumers. This answer choice is incorrect.

Get the most out of your LSAT Prep Plus subscription.

Analyze and track your performance with our Testing and Analytics Package.